LSAT and Law School Admissions Forum

Get expert LSAT preparation and law school admissions advice from PowerScore Test Preparation.

User avatar
 Dave Killoran
PowerScore Staff
  • PowerScore Staff
  • Posts: 5852
  • Joined: Mar 25, 2011
|
#44083
Complete Question Explanation
(The complete setup for this game can be found here: lsat/viewtopic.php?t=3405)

The correct answer choice is (E)

Answer choice (A) violates the third rule. Answer choice (B) is impossible because Y cannot respond to H. Answer choice (C) violates the inference made in the setup. Answer choice (D) cannot occur because then there would be no antibiotic for an organism to respond to, a violation of the first rule. Consequently, answer choice (E) is correct.

Get the most out of your LSAT Prep Plus subscription.

Analyze and track your performance with our Testing and Analytics Package.